Risk Final

अब Quizwiz के साथ अपने होमवर्क और परीक्षाओं को एस करें!

Shemar bought a building two years ago for $700,000. The building is insured under a commercial package policy with limits of $700,000 on the building. The policy is written on a replacement cost basis with an 80% coinsurance requirement and a $2,500 deductible. The current replacement cost of the building is $1 million. If a fire in the building causes damage that will cost $80,000 to repair, what amount will the insurance company pay? A)$67,500. B)$70,000. C)$77,500. D)$80,000.

)$67,500. RationaleThe formula for the recovery is:[(Amount of insurance carried/ Amount of insurance required) x Loss] - Deductible = Loss PaymentThe amount of insurance required is 80% x $1,000,000 = $800,000 [($700,000/$800,000) x $80,000] - $2,500 = $67,500

Joyce and Melvin have been married for 30 years. In 2020, they received $22,000 of Social Security benefits and had $12,000 of interest income. What portion of the Social Security benefit is taxable? A)$0. B)$6,000. C)$10,200. D)$11,000.

A)$0. Rationale The lesser of:50% of $22,000 = $11,000or0.5 [$12,000 + 0.5 ($11,000) - $32,000] < 0Since the answer calculated is less than $0, none of the Social Security benefits received by Joyce and Melvin are taxable.

Brad has purchased an umbrella liability policy requiring that he carry $300,000 per person as the liability limit in his personal auto policy. Brad's auto policy has split-limits of $100,000/$300,000/$50,000. In an auto accident caused by Brad, a pedestrian is injured and is awarded $250,000 by the court. How much will Brad's umbrella liability insurer pay in this situation? A)$0. B)$100,000. C)$150,000. D)$250,000.

A)$0. Rationale The umbrella liability insurer will settle the claim as if Brad had $300,000 of coverage under the personal auto policy, not the $100,000 that he did have. Therefore, the umbrella liability insurer will pay nothing.

Kathryn taught at Fun Academic University for 25 years and is filing for Social Security retirement benefits this year, when she turns age 72. Her PIA is $1,000 per month as adjusted for inflation. How much in benefits will she receive assuming her full retirement age was 66? A)$1,320. B)$1,360. C)$1,480. D)$1,587.

A)$1,320. Rationale Delayed benefits will increase at 8% per year of delay up until age 70. Therefore, her benefit will be 32% higher than her PIA. She has no increase for delaying beyond age 70 and the 8% increase is not compounded.

Mike has the following split limits of coverage on his Personal Auto Policy of 100/300/50. Which of the following best describes Mike's coverage? A)$100,000 per person for bodily injury, $300,000 per occurrence for bodily injury and $50,000 for property damage. B)$100,000 per covered auto, $300,000 per occurrence for covered auto and $50,000 for uninsured motorist. C)$100,000 per person for bodily injury, $300,000 per occurrence for property damage and $50,000 for uninsured motorist. D)$100,000 for property damage, $300,000 per person for bodily injury and $50,000 for property damage.

A)$100,000 per person for bodily injury, $300,000 per occurrence for bodily injury and $50,000 for property damage. RationaleSplits limits read: bodily injury / per occurrence for bodily injury / property damage

Under which of the following circumstances would the liability coverage of the car owner's PAP be available to the negligent driver?1. A neighbor borrows the insured auto for grocery shopping, believing this would be agreeable to the named insured because the neighbor had done the same thing several times previously.2. An eighteen-year-old college student, while intoxicated, takes a neighbor's car to show his buddies how he can start the car without an ignition key. A)1 only. B)2 only. C)Both 1 and 2. D)Neither 1 nor 2.

A)1 only. Rationale Option 1 is correct because the neighbor has reasonable belief that he or she is permitted to borrow the vehicle. Option 2 is not correct because the neighbor's eighteen-year-old son does not have reasonable belief that he is permitted to borrow the vehicle.

Which of the following statements concerning Medicare benefits available to those over 65 are true?1. Medigap coverage is available to cover costs not paid for by Medicare, but only under federal guidelines. 2. A patient pays deductibles for hospital stays, but nothing for doctor visits.3. Part A of Medicare coverage is optional for those covered by Social Security and requires a monthly premium. A)1 only. B)2 and 3. C)1 and 3. D)1, 2, and 3.

A)1 only. Rationale Part A (hospital coverage) covers all those eligible for Social Security, without any payment of premiums. Part B is optional and requires the payment of premiums. Medigap coverages come in only a limited number of combinations, mandated by federal regulations. These plans pay for various costs not covered by Medicare, such as copayments and the Part A deductible. Part A does require a deductible for hospital stays, but Part B does not fully cover routine doctor visits (other than a one-time "Welcome to Medicare" wellness visit and one annual wellness visit.

Kareem is a drug rep and planning on retiring next month. He is using his accumulated $200,000 to purchase an annuity. Which of the following options will give him the largest monthly annuity payment assuming his life expectancy is 20 years and his wife's life expectancy is 22 years. A)10 year term certain. B)Single life annuity over Kareem's life. C)100% joint life annuity over Kareem and his spouse's lives. D)75% joint life annuity over Kareem and his spouse's lives.

A)10 year term certain. Rationale Payments over 10 years will be larger than payments over 20 years or more, even if they are guaranteed.

Otto rents an apartment for $500 per month and has $50,000 content coverage. If he is unable to occupy his apartment due to a negligent fire caused by a neighbor, for up to how many months could he rent another apartment if the cost of the new apartment is $750 per month? A)60 months. B)6 months. C)None because negligent acts are not covered. D)None because content coverage does not cover reimbursement for rent.

A)60 months. Rationale 30% of content equals $15,000. If the excess rent is $250 per month, Otto could continue to receive loss of use benefits for up to 60 months if that amount of time was reasonable.

Which of the following is true? A)A fixed annuity mitigates the risk of superannuation. B)A fixed annuity mitigates the risk of superannuation and inflation. C)A fixed annuity is always a deferred annuity. D)A fixed annuity is always for a single life expectancy.

A)A fixed annuity mitigates the risk of superannuation. Rationale Annuities mitigate the risk of superannuation. Option b is incorrect as fixed annuities are subject to loss of purchasing power. Option c is incorrect as fixed annuity can either be deferred or immediate. Option d is incorrect as a fixed annuity can be for a single life, joint life, or for a guaranteed term.

Harry, age 63 purchased an immediate annuity. The annuity will provide monthly payments to Harry for as long as he lives. If he dies before receiving payments for 20 years, the remaining payments will go to his beneficiary. What type of annuity did Harry purchase? A)A life annuity with a term-certain guarantee. B)An installment refund annuity. C)A straight-life annuity. D)A joint and survivor annuity.

A)A life annuity with a term-certain guarantee. Rationale Harry purchased a life annuity with guaranteed payments. In this case, the insurer has promised to make at least 240 payments (20 years x 12 months).

Medicare Part A provides hospital coverage. Which of the following persons is not covered under Part A? A)A person 62 or older and receiving railroad retirement. B)Disabled beneficiaries regardless of age that have received Social Security for two years. C)Chronic kidney patients who require dialysis or a renal transplant. D)A person 65 or older entitled to a monthly Social Security check.

A)A person 62 or older and receiving railroad retirement. Rationale Medicare Part A requires a person to be age 65. People who are disabled or have permanent kidney failure are entitled to Medicare at any age.

Bert and Enrie are brothers who frequently drive each other's cars. Their automobiles are insured as follows: Bert is negligently driving Ernie's car and has an accident causing bodily injury to the other party involved in the accident in the amount of $50,000. Which insurer will pay, and how much? A)ABC will pay $40,000, and LMV will pay $10,000. B)LMV will pay $50,000, and ABC will pay nothing. C)ABC will pay $40,000, and LMV will pay nothing. D)ABC will pay $25,000, and LMV will pay $25,000.

A)ABC will pay $40,000, and LMV will pay $10,000. RationaleThe auto's insurance is primary, and the non-owner operator's insurance is secondary so, Ernie's policy will pay to its limits, then Bert's policy will pay up to its limits.

Colson wants to participate in the stock market by purchasing a deferred annuity but wants protection against the loss of his principal. What annuity is best suited for Colson? A)An equity-indexed annuity. B)A variable annuity. C)A fixed annuity. D)A single-premium, variable annuity.

A)An equity-indexed annuity. Rationale An equity-indexed annuity has the protection of guaranteed minimum rate of return, while still having the potential of earning higher equity returns. A fixed annuity will have a guaranteed rate of return, but will not have any participation in the stock market.

All of the following statements concerning Social Security benefits are correct except: A)In order to obtain SSI benefits, an individual must be age 65 or older and must be disabled. B)The number of days that Medicare covers care in hospitals and skilled nursing facilities is measured in what is termed benefit periods. C)The definition of disability is that the individual is unable to engage in any substantial gainful activity due to a physical or mental problem expected to last at least a year or expected to result in death. D)Benefits are payable at any age to workers who have enough Social Security credits and who have a severe physical or mental impairment that is expected to prevent them from doing "substantial" work for a year or more or who have a condition that is expected to result in death.

A)In order to obtain SSI benefits, an individual must be age 65 or older and must be disabled. Rationale In order to obtain SSI benefits, an individual must either be age 65 or older OR must be disabled.

With an equity-indexed annuity, there are several indexing methods. Which of the following is not one of them? A)Oscillation neutrality method. B)Annual reset (ratcheting method). C)High watermark method. D)Point-to-point method.

A)Oscillation neutrality method. RationaleChoices b, c, and d are all methods. Choice a is not a real indexing method.

Which of the following concerning the Social Security system is correct? A)SSI benefits are funded by the Treasury, not Social Security taxes, as are the other benefits. B)The Social Security retirement benefit is payable at full retirement age with reduced benefits as early as age 59 to anyone who has obtained at least a minimum amount of Social Security benefits. C)The two Medicare trust funds are the federal Medical Insurance Trust Fund for Part A and the Supplementary Hospital Insurance Trust Fund for Part B of Medicare benefits. D)Benefits can be paid to the dependent parents of a deceased insured worker at age 59 or over.

A)SSI benefits are funded by the Treasury, not Social Security taxes, as are the other benefits. Rationale The Social Security retirement benefit is payable at full retirement age with reduced benefits as early as age 62 to anyone who has obtained at least a minimum amount of Social Security coverage. The two Medicare trust funds are the federal Hospital Insurance Trust Fund for Part A and the Supplementary Medical Insurance Trust Fund for Part B of Medicare benefits. Social Security benefits can be paid to the dependent parents of a deceased insured worker at age 62 or over.

Which of the following statements concerning the homeowners series of policies is correct? A)Section II of the various homeowners policies covers personal liability and is identical for the entire series of homeowners policies. B)Both HO-02 and HO-03 cover losses caused by water damage resulting from seepage of water through basement walls. C)The inflation protection endorsement is available and provides that the amount of insurance is automatically increased for any increase in the Consumer Price Index. D)Homeowners Form HO-06 provides coverage for older properties, and losses are settled on an ACV basis.

A)Section II of the various homeowners policies covers personal liability and is identical for the entire series of homeowners policies. Rationale Option b is not a correct statement. None of the homeowners policies covers water damage caused by floods, backup through sewers or drains, or seepage through basement walls unless an endorsement is added. Option c is not a correct statement. The inflation guard endorsement provides for the dollar amount of insurance coverage to increase automatically every quarter. The increase is not tied to the Consumer Price Index, it is at a rate specified in the policy. Option d is not a correct statement. Form HO-08 provides this coverage - not HO-06.

Meg has money in a 401(k) plan with her employer. She has no dependents or heirs and wants to maximize her monthly income. She is retiring and wants to move the money into an annuity contract. Which would be the best option for Meg? A)Single-premium life annuity. B)Flexible-premium annuity. C)Deferred annuity. D)Installment refund annuity.

A)Single-premium life annuity. RationaleA single-premium life annuity because she wants the maximum income and she has no dependents or heirs.

Which of the following are correct? A)Surviving spouses are entitled to 100 percent of the worker's benefit amount after the worker dies. B)Divorced spouses can also claim benefits based on their ex-spouse's record if they were married 8 years or longer. C)For someone to delay benefits until age 70 based solely on payback or breakeven, the person would have to live to at least age 92, without considering the time value of money. D)All of the above are false.

A)Surviving spouses are entitled to 100 percent of the worker's benefit amount after the worker dies. Rationale Option a is correct. Option b is wrong because spouses have to be married 10 years, not 8. Option c is wrong because the age is closer to 82, not 92.

An insured with an HO-02 or an HO-03 might prefer specific insurance on certain types of personal property by means of a Scheduled Personal Property Endorsement for which of the following reasons? A)The HO-02 and HO-03 have sublimits applicable to certain types of personal property. B)The insurance rate for scheduled property is usually lower than for blanket coverage. C)The HO-02 and HO-03 do not cover loss caused by theft. D)These types of personal property are otherwise excluded from coverage.

A)The HO-02 and HO-03 have sublimits applicable to certain types of personal property. Rationale A Scheduled Personal Property Endorsement will not be subject to the sublimits that apply under the HO-02 and HO-03, so a loss will be covered for a higher value. The endorsement offers higher limits, elimination of the deductible, and open-perils coverage. Losses can be paid on a valued basis. Options b, c, and d are not correct statements.

Antoine immigrated from Italy last century, became a citizen and has worked the better part of his life in the United States, for which he is truly thankful. His full retirement age for Social Security benefits is age 66, but after a hard life working he wants to retire at age 63 and travel in America and back to his homeland. After contacting the Social Security administration, they informed him that his benefit at age 63 would be $1,200 per month. Just prior to retiring, he sold his business for $100,000. Which of the following statements is correct? A)The sale of the business will not impact the amount of his retirement benefits from Social Security. B)The sale of the business will cause his benefits to be reduced by $1 for every $2 over the annual limit. C)The sale of the business will definitely increase the portion of his Social Security benefits that are subject to income tax. D)The sale of the business may cause him to pay a higher Medicare Part B premium in the year of the sale due to his higher income.

A)The sale of the business will not impact the amount of his retirement benefits from Social Security. Rationale Option b is false. The sale is not earned income. Therefore, it will not result in a reduction of benefits. Option c is false because it might not increase the portion of the benefit subject to taxation ($100,000 is the sale price, not the taxable gain). 85 percent of his Social Security benefit may already be subject to taxation. Option d is false, as he would not receive Medicare until age 65; however, it is possible that his Medicare premium at age 65 will be increased due to the two year look-back for income.

A client contracted for a house to be built on a plot of land that cost him $100,000. The builder completed the house for $250,000. The client has contacted an insurance agent about obtaining an HO-03 homeowners insurance policy that will insure the house for its replacement cost. The policy will have an 80% coinsurance provision and a $2,000 deductible. How much insurance does the client need to buy to cover the full replacement cost of the house in the event of a total loss by fire? A)$200,000. B)$250,000. C)$280,000. D)$350,000.

B)$250,000. Rationale To provide full coverage for the replacement cost of the house, the client needs to buy a policy for the full replacement cost of $250,000. In the event of a total loss by fire, the land will not need to be replaced. The land should not be insured so coverage of $350,000 is not required. The house needs to be insured at the full replacement cost to pay for a total loss. If the client insures the house to only 80%, then the most the insurer will pay is 80% of the replacement cost. To get 100% replacement cost on the house, it must be insured for 100% of its replacement value which is $250,000.

Saul purchased a life insurance policy with a face value of $100,000. Unfortunately for Saul, and his wife, Rhea, he died racing "import cars." At the time of death, the life insurance policy had a cash value of $40,000. The beneficiary was his wife. Rhea, who has a life expectancy of 40 years, decided to take the single life annuity option with the life insurance proceeds. This option will pay her $500 per month for the rest of her life. How much of the monthly payment is taxable? A)$208. B)$292. C)$500. D)None, as life insurance proceeds are almost always tax-free to the beneficiary.

B)$292. RationaleExclusion ratio = investment / expected payments = $100,000 / (40 years x 12 months x $500) = 41.7%.Inclusion ratio = 1 - exclusion ratio = 58.3%Taxable amount = $500 x 58.3% or $292

Emile is single and received $28,000 of dividend income during the year. He also received $18,000 of Social Security benefits. What portion of his Social Security benefits are taxable? A)$0. B)$7,050. C)$9,000. D)$15,300.

B)$7,050. Rationale The lesser of: 85% of $18,000= $15,300 or 0.85 [$28,000 + 0.5 ($18,000) - $34,000]= $2,550 plus $4,500 equals $7,050

Antoine immigrated from Italy last century, became a citizen and has worked the better part of his life in the United States, for which he is truly thankful. His full retirement age (normal retirement age) for Social Security benefits is age 66, but after a hard life working he wants to retire at age 63 and travel throughout America and back to his homeland. If his benefit at age 66 is $1,000 per month, how much will he receive in Social Security retirement benefits if he begins receiving benefits at age 63? A)$700.00. B)$800.00. C)$812.50. D)$1,000.00. The same benefit, he will simply not receive it for as long.

B)$800.00. Rationale The benefit reduction for early retirement is 5/9ths of 1% for the first 36 months. If Antoine retires 3 years early at age 63, then his retirement benefit will be reduced by 20% to $800 per month.

The personal umbrella policy provides excess coverage over which of the following insurance?1. The personal auto policy liability coverage.2. The homeowners personal property coverage.3. The homeowners liability coverage. 4. The uninsured motorists coverage in the PAP. A)1 and 2. B)1 and 3. C)2 and 3. D)1, 2, 3, and 4.

B)1 and 3. Rationale The personal umbrella policy is primarily excess coverage for liability but does provide primary coverage for some exposures. The personal umbrella provides excess coverage for the liability coverage of the personal auto policy and for the homeowners liability coverage. It does not provide excess coverage for personal property insurance or uninsured motorists coverage.

In case of a loss covered under a property insurance policy, which of the following will be required?1. Notice of loss.2. Protection of the undamaged property.3. Proof of loss.4. Appraisal. A)1 and 2. B)1, 2, and 3. C)1, 3, and 4 D)2, 3, and 4.

B)1, 2, and 3. Rationale Item 4 is not correct because appraisal may not be necessary. Appraisal is required only when the parties cannot agree on the amount of the loss. Options 1, 2, and 3 are all required.

Betty Sue, age 75, is a widow with no close relatives. She is very ill, unable to walk, and confined to a custodial nursing home. Which of the following programs is likely to pay benefits towards the cost of the nursing home?1. Medicare may pay for up to 80 additional days of care after a 20-day deductible.2. Medicaid may pay if the client has income and assets below state-mandated thresholds. A)1 only. B)2 only. C)Both 1 and 2. D)Neither 1 nor 2.

B)2 only. Rationale Statement 1 is incorrect because Medicare covers all costs for the first 20 days of skilled nursing home care and covers the next 80 days with a deductible, provided that the nursing home stay occurs after at least a 3-day inpatient hospital stay.

Which of the following statements concerning the federal income tax treatment of monetary settlements is (are) correct?1. Under a structured settlement related to bodily injury, the portion of each payment received that represents interest is taxed as income, while that represented by principal is tax-free.2. Damages received because of employment discrimination are taxable as income.3. Punitive damages are tax-free for the recipient. A)1 only. B)2 only. C)1 and 3. D)2 and 3.

B)2 only. Rationale Statement 1 is incorrect because these payments are totally tax-free, both principal and interest. Statement 3 is incorrect because punitive damages are taxable as income to the recipient.

Which of the following parties to an annuity contract serves as the measuring life?1. Owner.2. Annuitant.3. Beneficiary. A)1 only. B)2 only. C)Both 1 and 3. D)Both 2 and 3.

B)2 only. RationaleThe annuitant's life is the measuring life for an annuitized payments in an annuity contract.

Which of the following concerning the Social Security system is correct? A)Workers entitled to retirement benefits can currently take early retirement benefits as early as age 59. B)A worker who takes early retirement benefits will receive a reduced benefit because he will receive more monthly benefit payments as payments commence earlier than if the worker had waited and retired at full retirement age. C)Family members of an individual who are eligible for retirement or disability benefits include a spouse if the spouse is at least 59 years old or under 59 but caring for a child under age 16. D)Generally, individuals who are over the age of 62 and receive Social Security benefits automatically qualify for Medicare benefits.

B)A worker who takes early retirement benefits will receive a reduced benefit because he will receive more monthly benefit payments as payments commence earlier than if the worker had waited and retired at full retirement age. Rationale Workers entitled to retirement benefits can currently take early retirement benefits beginning at age 62. Family members of an individual who is receiving retirement or disability benefits include a spouse if the spouse is at least 62 years old or under 62 but caring for a child under age 16. Generally, individuals who are age 65 or over and receive Social Security benefits automatically qualify for Medicare.

Section II of a HO-3 policy provides what type of protection for the homeowner? A)Dwelling. B)Damage to other's property. C)Loss of use. D)Personal property.

B)Damage to other's property. Rationale Section II deals with liability and medical payments to others. Options a, c, and d are in section I of the homeowners' policy.

Jennifer lives by herself and rents a one-bedroom apartment in Manhattan. Which of the following homeowners policies would be most appropriate for Jennifer to purchase? A)HO-3. B)HO-4. C)HO-5. D)HO-6.

B)HO-4. Rationale HO 4 is for renters.

All of the following statements concerning Social Security benefits are correct except: A)The maximum family benefit is determined through a formula based on the worker's PIA. B)If a worker applies for retirement or survivors' benefits before his 65th birthday, he must also file a separate application for Medicare. C)People who are disabled or have permanent kidney failure can get Medicare at any age. D)The Social Security Administration is concerned with beneficiaries' combined income, which, on the 1040 federal tax return, includes adjusted gross income and nontaxable interest income.

B)If a worker applies for retirement or survivors' benefits before his 65th birthday, he must also file a separate application for Medicare. Rationale If a worker applies for retirement or survivors' benefits before his or her 65th birthday, there is no need to file a separate application for Medicare.

Kim, a single 40-year-old, would like to invest in the stock market but wants her principal guaranteed against losses. What type of annuity is most suitable for Kim? A)A deferred, fixed annuity. B)A single-premium, variable annuity. C)A flexible variable annuity. D)An equity-indexed annuity.

D)An equity-indexed annuity. Rationale The equity-indexed annuity is the only option that meets her criteria. It will allow participation in the stock market while guaranteeing her principal against losses.

Raymond is turning 65 years of age and has been receiving Social Security benefits. Which of the following statements concerning his eligibility for Medicare benefits is correct? A)Raymond can sign up for Part A without Part B, but he cannot sign up for Part B later. B)Raymond will be enrolled automatically in Parts A and B. C)If Raymond signs up for Part B later, the premium will be the same as at age 65. D)If Raymond declines Part A and Part B, he will be able to sign up at any time after age 65.

B)Raymond will be enrolled automatically in Parts A and B. Rationale A person who is already receiving Social Security benefits when they reach age 65 will be automatically enrolled for Medicare unless they sign a form electing not to be covered. If a person enrolls in Part A, the person can enroll later in Part B, but only during specified periods each year. If a person enrolls during one of the general enrollment periods (January to March each year), coverage will become effective in July of that year. The premiums will increase for delayed enrollment.

Which of the following offers identical coverage for all forms of a homeowners insurance policy? A)Section I. B)Section II. C)No form is identical. D)All forms are identical.

B)Section II. RationaleLiability and medical payments coverage are identical in all HO policies.

When Artie was 45 years old, he invested $60,000 in an annuity. Ten years later, the value of the annuity was $180,000. He took a withdrawal from the annuity in the amount of $10,000. How is the withdrawal taxed? A)The $10,000 is taxable as ordinary income. B)The $10,000 is taxable as ordinary income and is subject to the 10 percent early withdrawal penalty. C)$6,666 of the $10,000 is taxable as ordinary income. D)$6,666 of the $10,000 is taxable as ordinary income and is subject to the 10 percent early withdrawal penalty.

B)The $10,000 is taxable as ordinary income and is subject to the 10 percent early withdrawal penalty. Rationale When distributions (withdrawals) are taken from an annuity before annuitizing, the entire distribution must be included in the taxpayer's gross income until all of the gain in the contract has been distributed. When taxable income is received from an annuity before the recipient (owner) reaches the age of 59½, a 10 percent penalty tax (10 percent of the taxable income) is imposed.

Which of the following statements correctly indicates a significant difference between HO-02 and HO-03? A)The dwelling and other structures coverage of the HO-02 is open-perils, while that of the HO-03 is named perils coverage. B)The dwelling and other structures coverage of the HO-03 is open perils, while that of the HO-02 is named perils coverage. C)The HO-02 is designed for a tenant and, therefore, does not cover the dwelling or other structures. D)The HO-02 is specifically designed for condominium owners.

B)The dwelling and other structures coverage of the HO-03 is open perils, while that of the HO-02 is named perils coverage. Rationale Option a is the reverse of the correct answer. Option c refers to HO-04. Option d refers to HO-06.

When determining the percentage of an annuity income that is not taxable, the investment in the annuity is divided by the total of the expected payments to be received. What is this quotient is called? A)The inclusion ratio. B)The exclusion ratio. C)The current ratio. D)The working capital ratio.

B)The exclusion ratio. Rationale The exclusion ratio equals the owner's investment in the annuity contract divided by the expected return on the annuity. The resulting percentage is multiplied by the distribution, or payment, received to calculate the portion of the payment that is not subject to income tax.

Part B of Medicare is considered to be supplemental insurance and provides additional coverage to participants. Which of the following is true regarding Part B coverage? A)The election to participate must be made at the time the insured is eligible for Part A Medicare and at no time after. B)The premiums for Part B are paid monthly through withholding from Social Security benefits. C)Once a participant elects Part B, he must maintain the coverage until death. D)Coverage under Part B does not include deductibles or coinsurance.

B)The premiums for Part B are paid monthly through withholding from Social Security benefits. Rationale Only option b is correct. Option a is incorrect because participation can occur after the initial eligibility. Participation is not required to be maintained for life, and Part B does have deductibles and/or coinsurance.

Automobile medical payments coverage may seem redundant to persons covered by adequate medical insurance. A major reason for purchasing automobile medical payments coverage when one has medical insurance is: A)To receive double payments if one is injured. B)To cover medical expenses for guests or passengers in the car. C)To increase one's chance of a lawsuit by injured passengers. D)To follow the principles of risk management and focus on the loss rather than on the effect.

B)To cover medical expenses for guests or passengers in the car. Rationale Option b is the answer, as spelled out in the policy.

Which of the following would be covered under the physical damage section of the PAP? A)Loss of any equipment designed for the reproduction of sound not permanently installed in the covered auto. B)Vandalism or malicious mischief damage to tires. C)Damage due to wear and tear. D)Loss to any custom furnishing or equipment in or upon any pickup, panel truck, or van.

B)Vandalism or malicious mischief damage to tires. Rationale The physical damage coverage is "all-risks" or open-perils coverage. Therefore, since option b is not one of the exclusions, it is covered. Options a, c, and d are excluded under the physical damage coverage of the PAP.

Sonny and Cher have their home insured under a HO-3 policy. The amount of the coverage on the dwelling, $100,000, has been left unchanged for several years. Yesterday, the home was struck by lightning, causing $10,000 of damage, as measured by actual cash value, and $18,000 of damage, as measured by replacement cost. The replacement cost of the home at the time of the lightning damage is estimated at $150,000. In this situation, which of the following amounts will Sonny and Cher recover from the insurer for this loss (ignoring any deductible)? A)Nothing, because the insurance is less than 80% of replacement cost. B)$10,000, the actual cash value of the damage. C)$15,000, five-sixths of the replacement cost of the damage. D)$18,000, the replacement cost of the damage.

C)$15,000, five-sixths of the replacement cost of the damage. Rationale If the amount of insurance carried is less than 80% of the dwelling's replacement cost at the time of the loss, the insurer will pay the greater of the ACV of the loss or the amount produced by this formula:[(Amount of insurance carried/ Amount of insurance required) x Loss] - Deductible = Loss PaymentHere, [$100,000 ÷ (80% x $150,000)] x $18,000 = $15,000. Since this exceeds the $10,000 ACV of the loss, $15,000 will be paid.

A CFP® professional has been consulted on the amount of payment the client can expect from an insurance claim. The client owns a commercial building that has a replacement value of $1,500,000. The building was insured for $1,000,000, and the policy contained an 80% coinsurance clause and a $4,000 deductible. The building had a fire that has been estimated to have resulted in a $350,000 loss. What should the client expect to receive in payment for this loss? A)$229,333. B)$233,333. C)$287,667. D)$291,667.

C)$287,667. Rationale This question requires you to use the coinsurance formula for calculating payments on insured losses. The insured has not purchased adequate limits of so a coinsurance penalty applies. The coinsurance formula is as follows:[(Amount of insurance carried/ Amount of insurance required) x Loss] - Deductible = Loss PaymentThe amount of the insurance carried was $1 million, and the building was valued at a replacement cost of $1,500,000. The insurance required was 80% of $1.5 million or $1.2 million. The insurance payment for the partial loss will be reduced because the insured did not carry the required amount of insurance. The percentage of the loss that will be utilized is calculated by dividing the $1 million of insurance carried by the $1.2 million of required insurance. The result of this division is then multiplied times the loss of $350,000, and the deductible of $4,000 is then subtracted.So using the formula we get the following result:[($1,000,000/$1,200,000) x $350,000] - $4,000 = $287,667

Dermott's personal auto policy has split liability limits of 100/300/25. In an auto accident where Dermott was at fault, four people in the other car were injured. The court awarded $125,000 to each of them for bodily injury, plus $30,000 for property damage to the other car. Also, the cost of Dermott's legal defense totaled $18,000 in lawyers' fees. In this case, how much of the $548,000 will be borne by Dermott's insurance company? A)$318,000. B)$325,000. C)$343,000. D)$418,000.

C)$343,000. Rationale The $300,000 limit is the maximum the insurer will pay for all bodily injuries in an accident, regardless of how many are injured. The $25,000 limit is the maximum the insurer will pay for property damage. The defense costs are covered in addition to the policy's limits.$300,000 + $25,000 + $18,000 = $343,000

Mason, who is now 70 years old, was a diligent saver during his working years and accumulated $100,000 at age 60. He used his savings to purchase a single premium annuity, which pays him $1,000 per month. If his life expectancy was 25 years when he purchased it, how much of each payment is subject to tax? A)$333. B)$555. C)$667. D)$1,000.

C)$667. Rationale Exclusion ratio = $100,000 / (25 years x 12 month x $1,000) = 33.33%Taxable portion = 1- exclusion ratio = 66.67%Taxable portion of the payment = $667

Hugh purchased a single premium annuity for $200,000. The annuity pays him $1,000 per month. If his life expectancy was 20 years when he purchased it, how much of each payment is not subject to tax? A)$177. B)$475. C)$833. D)$1,000.

C)$833. RationaleExclusion ratio = $200,000 / (20 years x 12 month x $1,000) = 83.33%Taxable portion = 1- exclusion ratio = 17.67%Non-taxable portion of the payment = $833

Clara purchases an annuity with after-tax dollars and wants payments to begin next month. What type of annuity did Clara purchase?1. Immediate Annuity.2. Deferred Annuity.3. Qualified Annuity.4. Nonqualified Annuity. A)1 and 3. B)2 and 3. C)1 and 4. D)2 and 4.

C)1 and 4. Rationale The payments are not deferred, and the annuity is nonqualified because it was purchased with after-tax dollars. Immediate annuities usually begin paying one month after (and must begin within one year, such as when annual payments are selected rather than monthly).

Which of the following statements are true as to Supplemental Security Income (SSI) benefits?1. Supplemental Security Income makes monthly payments to individuals with low income and few assets.2. The basic monthly benefit for Supplemental Security Income in 2020 is $783.00 for one person and $1,175 for married couples.3. The asset determination for Supplemental Security Income eligibility does not include the value of one's home or car.4. The definition of disability is satisfied when the individual is unable to engage in any substantial gainful activity due to a physical or mental problem expected to last at least three years or expected to result in death. A)1 and 3. B)2 and 4. C)1, 2 and 3. D)1, 2, 3 and 4.

C)1, 2 and 3.Rationale All of the statements are true except statement 4. Statement 4 is in reference to Social Security Disability Income (SSDI), and would be correct (for SSDI) if the disability was expected to last at least one year, not three years as provided.

Selena is retiring in the next month. She was the bread winner in the family while Rafeal was the stay-at-home dad. They have limited assets but will receive Social Security. Which of the following is the best type of annuity for her to select? A)A 20-year term certain annuity. B)A straight life annuity on her life. C)A joint-life annuity on the lives of Selena and Rafeal. D)A joint-life annuity on the lives of Selena and Rafeal with a 10-year term certain.

C)A joint-life annuity on the lives of Selena and Rafeal. Rationale The best option is an annuity over both of their lives since they do not have a lot of other assets. The question does not mention whether Rafeal has a skill, or the ability and desire, to go find work. A 10-year term certain option on the joint-life annuity will make the payments go down.

Drake is a financial planner whose client, Bernice, is a widow. Bernice is 67 years old and about to retire, but is concerned about outliving her money since her parents both lived into their 90's. Drake has discussed the idea of a life annuity with Bernice, but she is concerned that if she were to die sooner than expected, the amount she invested might be lost. She would prefer an investment that would at least provide something for her grandson if she were to die in a short period of time without receiving back her total premiums. Which of the following is an appropriate recommendation for Drake to make to Bernice? A)A 10-year period certain annuity. B)A hybrid annuity/long-term care policy. C)A life annuity with cash refund. D)Drake should not recommend annuities because experts such as Suze Orman say they are poor investments.

C)A life annuity with cash refund. Rationale A life annuity with cash refund will guarantee Bernice receives payments for the rest of her life, no matter how long she lives, but if she dies before receiving back an amount equal to her premium payments, the difference between her total premiums and the amount she received will be paid as a lump sum to her beneficiary. The 10-year period certain annuity will not provide a guaranteed life income for Bernice so it would not be appropriate. A hybrid annuity and long-term care policy is not the best recommendation because Bernice has not stated a need for long term care coverage and has not asked about long-term care policies. Option d is incorrect because, like all investments, annuities are suitable for some and unsuitable for others. The client's individual situation will determine whether an annuity is a good investment for them.

Patrick is age 67 and receives Social Security retirement income that covers 60 percent of his monthly expenses. He has no dependents. He would like to invest $200,000 in an annuity that will mitigate inflation and provide him with the highest monthly income. Although he has a 20-year table life expectancy, he thinks he has a much longer life expectancy. Which annuity is most suitable for Patrick? A)A single-premium annuity. B)A deferred, fixed annuity. C)A single-premium, variable annuity. D)A single-premium, variable annuity with a guaranteed term equal to his table life expectancy.

C)A single-premium, variable annuity. Rationale The variable annuity will provide inflation protection, and any guaranteed term will reduce the monthly benefit. He needs an immediate annuity, not a deferred annuity, since he is retired and attempting to supplement his income.

Pedro was seriously injured in an auto accident and was awarded $750,000 in damages. Instead of accepting the money in a lump sum, Pedro's attorney worked out with the defendant's attorney an arrangement under which Pedro would receive the following amounts:* $100,000 in cash now* An income of $10,000 per month, starting after one month and continuing for 5 years* A lump-sum payment of $350,000 at the end of the fifth year, concurrent with the final $10,000 income checkThe arrangement worked out between the two sides in this case is called: A)A variable annuity. B)A pure life annuity. C)A structured settlement. D)A like-kind exchange.

C)A structured settlement. Rationale Options a and b are incorrect because although the settlement includes an annuity, the annuity is not the entire settlement. Option d is an income tax planning technique that is unrelated to structured settlements.

Which of the following statements concerning professional liability insurance is correct? A)Errors and omissions coverage protects the insured for liability for having negligently caused mental anguish, as well as bodily injury. B)Lawyers and accountants are more likely to purchase malpractice coverage rather than errors and omissions coverage. C)A surgeon is more likely to purchase malpractice insurance rather than errors and omissions coverage. D)A pharmacist is more likely to purchase errors and omissions insurance rather than malpractice coverage.

C)A surgeon is more likely to purchase malpractice insurance rather than errors and omissions coverage. Rationale Malpractice coverage is typically purchased by those whose negligence is likely to cause bodily injury. A surgeon is in this category. Option a is not a correct statement. Errors and omissions coverage protects the insured for those liabilities that do not include bodily injury. Option b is not a correct statement. Lawyers and accountants are more likely to purchase errors and omissions coverage. The negligence of lawyers and accountants is not likely to give rise to bodily injuries. Option d is not a correct statement. A pharmacist is more likely to purchase malpractice coverage. Clearly, negligence on the part of a pharmacist could result in bodily injury to a person whose drug prescription was incorrectly filled.

Which of the following statements are true for the PIA of individuals who first become eligible for retirement benefits in 2020?1. Their PIA is 90% of the first $960 of their AIME, plus other amounts.2. Their PIA is 32% of their AIME over $960 up to $5,785, plus other amounts.3. Their PIA is 15% of their AIME that exceeds $5,785, plus other amounts. A)1 and 2. B)1 and 3. C)All of the above statements are true. D)None of the above are true.

C)All of the above statements are true. Rationale For individuals who first become eligible for retirement benefits or disability insurance benefits in 2020 or who die in 2020 before becoming eligible for benefits, their PIA will be the sum of: 90 percent of the first $960 of their AIME, plus 32 percent of their AIME over $960 up to $5,785, plus15 percent of their AIME that exceeds $5,785.(Maximum PIA for 2020 is $3,011)

There are many options regarding the type of annuity one can purchase. Which of the following is correct? A)A fixed annuity will have the initial rate, which lasts until the end of the first life. B)A variable annuity always has funds invested in equity subaccounts. C)An equity-indexed annuity generally has less risk than a variable annuity. D)Earnings within fixed, variable and equity-indexed annuities are never subject to income tax until annuitization occurs and distributions begin.

C)An equity-indexed annuity generally has less risk than a variable annuity. Rationale Option a is false as the initial rate will last for specified period of time, not for a life. Option b is false as funds in a variable annuity can be invested in a variety of subaccount types, including cash, bonds, and equities. Option c is correct, as equity-index annuities have more guaranteed rates of return. Option d is not correct as earnings will be subject to tax upon withdrawals from the annuity, such as before annuitization.

Arline is a 70-year-old widow with no dependents. She wants to invest in an annuity that will produce income now. She has $100,000 to invest and wants to receive the most she can in monthly income. Which of the following is the most suitable annuity for Arline based on her objectives? A)A longevity annuity. B)A 20-year term certain, fixed annuity. C)An immediate, single-premium life annuity. D)A deferred, fixed annuity.

C)An immediate, single-premium life annuity. Rationale Arline wants the greatest amount of income now. A single-life annuity with no guarantee will provide her with the highest monthly income. A longevity annuity is usually deferred and any guarantees or use of a joint-life expectancy will reduce the monthly benefit.

Which of the following statements concerning professional liability insurance coverage on an occurrence basis or a claims-made basis is correct? A)An occurrence basis policy covers claims arising from events that occur during the policy period, and a claim is filed during the same policy period. B)Claims-made policies cover all claims regardless of when the claim is made. C)An insured who changes from a claims made policy to an occurrence basis policy, or who retires, will need to purchase a "tail" (extended reporting period) on the claims made policy to ensure adequate protection. D)An insured who changes from an occurrence basis policy to a claims made policy will need to purchase a "tail" (extended reporting period) on the occurrence basis policy to ensure adequate protection.

C)An insured who changes from a claims made policy to an occurrence basis policy, or who retires, will need to purchase a "tail" (extended reporting period) on the claims made policy to ensure adequate protection. Rationale For an occurrence basis policy, coverage applies if the event occurs during the policy period; the claim may be filed anytime. Therefore, option a and d are incorrect. Option b is not a correct statement. For a claims-made policy, the claim must be filed during the policy period, regardless of when the loss-causing event occurred.

For which of the following persons will medical expenses be paid under the Medical Payments Coverage of the personal auto policy? A)Any person other than the insured or family member injured in an accident with the covered auto. B)Any pedestrian struck by the covered auto. C)Any insured or family member injured in an accident while occupying any auto. D)Any person injured by a vehicle not designed for use on public roads.

C)Any insured or family member injured in an accident while occupying any auto. Rationale The Medical Payments Coverage of the personal auto policy will pay for the medical expenses of any insured or family member injured in an accident while occupying any auto. The coverage does not pay for the medical expenses of persons other than the insured or family because the liability coverage will pay for those expenses. Note that the Medical Payments to Others coverage of the homeowners policy is different in that it pays only medical expenses for persons other than the insured.

Mary lives in Idaho where she carries the state-mandated minimum liability insurance on her car (15/30/10) through her personal automobile policy (PAP). She is driving through Oklahoma and has a wreck with Gerri. Oklahoma requires minimum liability insurance of 30/50/20. Gerri suffers bodily injury in an amount of $40,000 and Gerri's vehicle is damaged in an amount of $22,000. How much will Gerri collect from Mary's PAP policy? A)Bodily injury $40,000 and property $22,000. B)Bodily injury $15,000 and property $10,000. C)Bodily injury $30,000 and property $20,000. D)Bodily injury $30,000 and property $22,000.

C)Bodily injury $30,000 and property $20,000. Rationale Mary's policy limits will increase to the amount required in Oklahoma. Gerri will have to sue Mary for her excess non-insured losses.

Annuities provide many benefits and have many advantages over other alternative types of investments. Which of the following is not an advantage of a single life annuity? A)The annuity mitigates the risk of superannuation. B)Earnings within the annuity are tax deferred. C)Distributions receive favorable capital gain treatment. D)The owner's estate excludes the value of the annuitized life annuity.

C)Distributions receive favorable capital gain treatment. Rationale Options a, b and d are correct. Option c is incorrect as annuity payments are treated as ordinary income and not capital gain income.

Luther owns a life insurance policy. He decided that he no longer has a need for life insurance and wants to exchange it for an annuity. To get the annuity he wants, he will have to exchange the life policy and add additional money. Which of the following is correct? A)He can make the exchange, but it will be taxable to the extent of the cash value. B)He can make the exchange, but it will be taxable to the extent of the cash value less the additional money he puts into the annuity. C)He can make the exchange, which will not be taxable. D)He can make the exchange, which will not be taxable, but his basis will not reflect any of the investment into the life insurance policy.

C)He can make the exchange, which will not be taxable. Rationale An exchange of a life policy to an annuity is a tax-free exchange under Section 1035. The basis will include the original investment and the additional funds.

Ted just retired and has several sources of income. He has Social Security and a pension from his employer, HTD Enterprises. Social Security is adjusted for inflation, but his pension is not. Ted is concerned about outliving his funds. What might he purchase to provide protection against running out of money late in retirement? A)Immediate variable annuity. B)Deferred fixed annuity. C)Longevity annuity. D)Life insurance.

C)Longevity annuity. Rationale A longevity annuity can be purchased relatively inexpensively and can provide benefits later in retirement, such as at age 85. The immediate variable annuity does not help with his needs, nor does the deferred fixed annuity.

Raymond lived in New Orleans during hurricane Katrina. His home was destroyed by wind damage, and he was forced to live in a hotel for three months. Which section of his homeowners policy would reimburse him for the increased living expenses associated with the hotel stay? A)Dwelling. B)Other structures. C)Loss of use. D)Personal property.

C)Loss of use. RationaleSection 1, Coverage D, loss of use will only pay for the increases in living expenses in excess of what he used to pay.

All of the following statements are correct regarding a Personal Auto Policy Part C (Uninsured Motorists) coverage except: A)Payment for property damage. B)Payment for lost wages. C)Payment for punitive damages. D)All of the above.

C)Payment for punitive damages. RationaleThe PAP uninsured motorist does not pay for punitive damages.

All of the following statements are correct regarding a Personal Auto Policy Part B (Medical Payments) coverage except: A)Provides payment for medical expenses of an insured due to an auto accident. B)Provides medical payments if struck as a pedestrian. C)Provides payments for medical expenses for household pets if struck by the covered auto. D)Provides payment for medical expenses of anyone occupying the insured's covered auto.

C)Provides payments for medical expenses for household pets if struck by the covered auto. RationaleA PAP does not cover pets.

Social Security is funded through all of the following except: A)Employee payroll tax. B)Employer payroll tax. C)Sales tax. D)Self-employment tax.

C)Sales tax. Rationale Employee and employer payroll tax and self-employment tax are the sources of funding for Social Security. Sales tax does not fund Social Security.

Which of the following risks can an annuity mitigate? A)Superannuation. B)Mortality. C)Superannuation and purchasing power. D)Mortality and purchasing power.

C)Superannuation and purchasing power. Rationale Annuities can help lessen superannuation (risk of outliving funds) and purchasing power (if the annuity is variable). An annuity does not reduce the likelihood of dying (mortality).

Which of the following personal property losses would not be fully covered (less any deductible) by an unendorsed HO-3 policy? A)The insured's golf clubs, worth $1,300, were stolen from the trunk of his locked car by someone who forced open the trunk. B)The insured's hunting rifles, worth $1,250, were stolen from his tent during a hunting trip. C)The insured's laptop computer, worth $2,400, was taken from her while she was using it during lunch at a restaurant. D)The insured's canoe, worth $900, was taken from his garage.

C)The insured's laptop computer, worth $2,400, was taken from her while she was using it during lunch at a restaurant. Rationale Electronic property has a limit of $1,500.

Perry, who is 50 years old, was building a new home for his family. However, he was running out of money and could not afford the pool they fell in love with. Since his family was upset, he decided to take a withdrawal from his annuity. He had contributed $100,000 to the annuity, and the value of the annuity today is $300,000. He decided to take a withdrawal of $60,000 from the annuity. Which of the following is correct? A)$40,000 is taxable as ordinary income. B)$40,000 is taxable as ordinary income and subject to the early withdrawal penalty. C)$60,000 is taxable as ordinary income. D)$60,000 is taxable as ordinary income and subject to the early withdrawal penalty.

D)$60,000 is taxable as ordinary income and subject to the early withdrawal penalty. Rationale When distributions (withdrawals) are taken from an annuity before annuitizing, the entire distribution must be included in the taxpayer's gross income until all of the gain in the contract has been distributed. When taxable income is received from an annuity before the recipient (owner) reaches the age of 59½, a 10 percent penalty tax (10 percent of the taxable income) is imposed.

Diamond, who has a standard HO-3 policy, rents a room in her house to a college student. In the event that the house is seriously damaged by a fire or other peril, which of the following losses, in addition to the actual cash value of the damaged home, is Diamond likely to encounter:1. Lost income.2. Extra living costs.3. Extra costs to replace the damaged personal property. A)1 and 2. B)1 and 3. C)2 and 3. D)1, 2, and 3.

D)1, 2, and 3. Rationale Diamond is likely to lose the rental income from the student and to incur extra costs for alternate housing and restaurant meals. Also, the replacement cost is likely to exceed the ACV of some of the damaged personal property

The other-than-collision coverage in a PAP will cover losses from which of the following causes?1. Vandalism.2. Impact with a bear.3. Impact with a telephone pole.4. Falling objects. A)1 and 3. B)2 and 3. C)2 and 4. D)1, 2, and 4.

D)1, 2, and 4. Rationale Other-than-collision coverage will pay for losses from vandalism, contact with an animal, and falling objects. Impact with a telephone pole is a collision.

The HO-03 provides medical payments coverage for which of the following persons?1. The insured and family members residing with him or her.2. Any person on the insured premises with the permission of any insured.3. Any person injured away from the insured premises if the injury is caused by the activities of the insured.4. Any person injured away from the insured premises if the injury is caused by an animal owned by or in the care of any insured. A)1 and 2. B)3 and 4. C)1, 2, and 3. D)2, 3, and 4.

D)2, 3, and 4. Rationale Options 2, 3, and 4 are all covered under the medical payments coverage of the HO-03. Option 1 is not covered. The medical payments coverage provided by the HO-03 is not designed to provide health insurance coverage for the insured and his or her family.

Brisco, now deceased, was married for 12 years. He had two dependent children, ages 10 and 12, who are cared for by their mother age 48. Brisco's mother, age 75, was his dependent and survived him. At the time of his death, he was currently but not fully insured under Social Security. His dependents are entitled to all of the following benefits except: A)A lump-sum death benefit of $255. B)A children's benefit equal to 75% of Brisco's PIA. C)A caretaker's benefit for the children's mother. D)A parent's benefit.

D)A parent's benefit. Rationale A lump-sum death benefit of $255 is payable to the surviving spouse or children of the deceased worker if he was fully or currently insured. The children's benefit is payable because Brisco was either currently or fully insured. It is 75% of his PIA. The children's mother would be entitled to a benefit for caring for the children under the age of 16. His dependent mother is only entitled to a benefit if he was fully insured, not currently insured.

Resident family members are covered for the purposes of Part A (Liability Coverage) for a Personal Auto Policy when operating all of the following except: A)Their auto. B)A rented auto. C)A borrowed auto. D)A replacement auto after 31 days.

D)A replacement auto after 31 days. Rationale You have a duty to notify the insurance company within first 30 days of purchasing a replacement car. A replacement car means you are exchanging the insurance on the old car for the new car, not adding an additional car.

Neil is 67 years old and about to retire. His table life expectancy is 20 years. He has $25,000 in a retirement plan for which there is no lump-sum withdrawal option. Neil has 2 children and wants to make sure he receives all of the $25,000 and that he can leave his other assets to his children. He wants to participate in equity returns. Which is the most suitable for Neil? A)A single-life annuity. B)A joint-life annuity with his children. C)A single-life, fixed annuity with a term certain of 20 years. D)A single-life, variable annuity with a term certain of 20 years.

D)A single-life, variable annuity with a term certain of 20 years. Rationale The variable annuity will allow him to participate in equity returns, and the guarantee period equal to his life expectancy will likely assure him of getting his entire principal back.

All of the following statements concerning the Social Security system are correct except: A)If a worker receives retirement benefits based on his own earnings record, the worker's retirement benefits will continue whether married or divorced. B)Widows and widowers, whether divorced or not, will continue to receive survivors' benefits upon remarriage if the widow or widower is age 60 or older. C)By providing the name of a country or countries to be visited and the expected departure and return dates, the Social Security Administration will send special reporting instructions to the beneficiaries and arrange for delivery of checks while abroad. D)A special one-time payment of $1,000 may be made to a deceased worker's spouse or minor children upon death.

D)A special one-time payment of $1,000 may be made to a deceased worker's spouse or minor children upon death. RationaleA special one-time payment of $255 may be made to a deceased worker's spouse or minor children upon death.

Which of the following statements about annuities is not correct? A)Term certain and joint life terms are not mutually exclusive. B)Variable annuities can be invested in subaccounts that invest in fixed income securities. C)The annuity benefit payment for a 20-year term certain will be more than the annuity payment for a single life annuity with a 20-year term certain option for an annuitant whose life expectancy is 20 years. D)All annuities are subject to minimum distribution rules.

D)All annuities are subject to minimum distribution rules. Rationale Option a is a true statement as the options can be combined. Option b is a true statement as the annuity owner chooses to invest in a variety of subaccount types, including fixed income. Option c is a true statement as the insurance company is more likely to pay more with the life annuity term certain combined option. Option d is false - only qualified annuities are subject to minimum distribution rules.

Antoine immigrated from Italy last century, became a citizen and has worked the better part of his life in the United States, for which he is truly thankful. His full retirement age for Social Security benefits is age 66, but after a hard life working he wants to retire at age 63 and travel in America and back to his homeland. Antoine sold his restaurant for $150,000 and was retained by the acquiring company as a consultant for a one-year period. His consulting fee this year will be $50,000 and the benefit he expects to receive from Social Security is $2,000 per month. Which of the following statements is correct? A)Antoine should delay receiving Social Security retirement benefits until after this year because he will be in a high tax bracket due to the sale of his business. B)Antoine should delay receiving Social Security retirement benefits until after this year because there will be such a large reduction in benefits due to his consulting income and the sale of the business. C)Antoine should begin receiving Social Security benefits today because, although there is a reduction in benefits today, there will be an increase after full retirement age. D)Antoine would be better off beginning Social Security benefits next year.

D)Antoine would be better off beginning Social Security benefits next year. Rationale Option a is not the best answer - however, Antoine is likely to be in a higher tax bracket because of the sale and theconsulting income. Option b is not correct because the sale of the business is not earned income and will not cause in a reduction of benefits. His consulting income is earned income. Option c is not correct - benefits reduced due to retirement before FRA are permanently reduced, although any benefit reduction due to the earnings test will be offset by an increase in benefits after full retirement age. Option d is correct as his earned income will be lower and his tax bracket will likely be lower.

Which of the following would not be considered an insured person for the purposes of Part A (Liability Coverage) for a Personal Auto Policy? A)You. B)Any family member. C)Any person using "your covered auto" with permission. D)Any person using "your covered auto" without permission.

D)Any person using "your covered auto" without permission. RationaleA person using an automobile without permission is not covered.

All of the following statements are correct regarding a Personal Auto Policy Part D (Coverage for Damage to Your Auto) coverage except: A)Collision applies when your car hits another vehicle. B)Comprehensive covers fire, theft, or vandalism. C)Collision covers damages to a borrowed or rented vehicle. D)Collision coverage includes contact with an animal or bird.

D)Collision coverage includes contact with an animal or bird.

There are several types of annuities. Which of the following orders the investment options from least risky to most risky? A)Fixed, variable, equity-indexed. B)Variable, fixed, equity-indexed. C)Equity-indexed, variable, fixed. D)Fixed, equity-indexed, variable.

D)Fixed, equity-indexed, variable. RationaleThe most conservative investment is fixed, followed by equity-indexed and then variable.

Quincy, age 33, plans to retire at age 67. Quincy is a consultant and his income varies widely on a monthly basis. Quincy wants to invest in an annuity over his work life expectancy. Which of the following annuities is most suitable for Quincy? A)Fixed premium, immediate annuity. B)Single premium, immediate annuity. C)Fixed premium, deferred annuity. D)Flexible premium, deferred annuity.

D)Flexible premium, deferred annuity. Rationale Quincy needs a deferred annuity because he wants to invest over his work life expectancy and he needs a flexible premium because of his variable income.

Which of the following statements concerning specific HO forms is correct? A)HO-08 is designed to cover the special exposures of a condominium unit owner. B)HO-02 provides "all-risks" coverage for the dwelling and named perils coverage for the dwelling's contents. C)If the premium were the same, a rational homeowner would prefer to have the coverage of HO-02 rather than HO-03. D)HO-04 provides no coverage on a dwelling or on other structures.

D)HO-04 provides no coverage on a dwelling or on other structures. Rationale HO-04 is designed for tenants; it provides only specified-perils coverage for contents. Option a is not a correct statement. The HO-06 is designed for condominium owners. Option c is not correct. If the premium were the same, a homeowner would prefer an HO-03, rather than an HO-02. The HO-03 provides open perils coverage on the dwelling. The HO-02 provides only named perils coverage on the dwelling.

Hector, who has retired to Miami, decided to purchase a condominium unit on the beach. Which of the following homeowners policies would be most appropriate for Hector to purchase? A)HO-3. B)HO-4. C)HO-5. D)HO-6.

D)HO-6. RationaleHO 6 is for condo owners.

Cody decided to purchase a variable annuity and invested $50,000. He made a few poor investment decisions and the market dropped. Like many investors, he decided to sell when the value was low. He surrendered the annuity and received $27,000, after the $3,000 surrender fee. Which of the following is true? A)The loss is not deductible. B)He can take an itemized deduction of $23,000. C)He can take a capital loss of $20,000. D)He can take an ordinary loss of $20,000.

D)He can take an ordinary loss of $20,000. Rationale With the surrender of an annuity for a loss, the owner can take a loss that does not include the surrender charge. Thus, option b is not correct. It is not a capital loss. Taking the loss as an ordinary loss is more aggressive, but there is some authority for the position.

Robin Elizabeth qualifies for a retirement benefit of $250 and a spouse's benefit of $400. At her full retirement age, she will receive which of the following? A)Both $250 and $400. B)The higher of her benefit or her spouse's benefit. C)Her spouse's benefit of $400. D)Her benefit of $250 plus $150 from her spouse's benefit.

D)Her benefit of $250 plus $150 from her spouse's benefit. Rationale She will receive her benefit plus an additional amount due to the higher spousal benefit.

Mig purchased a straight life annuity from Confident Results Annuity Provider using funds from his 401(k) plan. Which of the following is correct? A)His annuity will make payments for his life and that of his beneficiary. B)His annuity cannot be a variable annuity. C)His annuity will be included in his gross estate at death. D)His annuity will have no return of basis.

D)His annuity will have no return of basis. Rationale Option a is false as a straight life annuity will pay for only one life. Option b is false as it could be a variable annuity. Option c is false as there will be no payments to include in his gross estate since the annuity ends at his death. Option d is correct as pre-tax funds were used to purchase the annuity. Therefore, all the annuity payments will be taxable.

Which of the following statements concerning types of annuities is not correct? A)In a deferred annuity, if the insured dies during the accumulation period, a death benefit is payable. B)In an equity-indexed annuity, the principal may be guaranteed, while interest credited is based on the performance of the stock market. C)In a cash refund annuity, benefit payments will at least equal the purchase price of the contract. D)In a variable annuity, the value of the accumulation units is fixed, while the value of the annuity units during the liquidation period fluctuates.

D)In a variable annuity, the value of the accumulation units is fixed, while the value of the annuity units during the liquidation period fluctuates. Rationale The value of both the accumulation units and the annuity units fluctuates.

All of the following are requirements for Qualified Longevity Annuity Contracts (QLACs), EXCEPT? A)A maximum of $135,000 (in 2020) may be used to purchase the QLAC. B)A QLAC is purchased in an IRA or defined contribution retirement plan. C)In order to be qualified, a QLAC must be annuitized no later than age 85. D)In order to be qualified, a QLAC must be a variable annuity.

D)In order to be qualified, a QLAC must be a variable annuity. Rationale Variable and indexed annuities are not permitted to be QLACs. All of the other statements are true regarding the rules for Qualified Longevity Annuity Contracts.

At the age of 40, Kennedy deposited $50,000 in a nonqualified, single-premium deferred annuity. Ten years later, she surrenders the contract for a lump-sum distribution of its $100,000 value. Which of the following statements is correct? A)Kennedy will owe taxes on $50,000. B)Kennedy will owe taxes on the full $100,000. C)Kennedy will owe taxes and a 10% penalty on the full $100,000. D)Kennedy will owe taxes and a 10% penalty on $50,000.

D)Kennedy will owe taxes and a 10% penalty on $50,000. Rationale Taxes were paid on the principle and the interest (growth) is taxed as income. Therefore, Kennedy will owe taxes on the $50,000 of earnings in the contract. Since she is under the age of 59½, she will also owe a penalty on the $50,000 of taxable earnings.

The personal liability coverage of the homeowners series covers bodily injury and property damage liability in which of the following situations?1. The liability arises out of the rendering of professional services.2. The liability arises out of the insured's use of a large motorboat (owned by the insured) powered by an inboard motor.3. The liability arises out of the ownership of an aircraft. A)2 only. B)1 and 3. C)1, 2, and 3. D)Neither 1, 2, nor 3.

D)Neither 1, 2, nor 3. Rationale Liability coverage is not provided for any of these activities.

A person receiving Social Security benefits under full retirement age can receive earned income up to a maximum threshold without reducing Social Security benefits by the earnings test. Which of the following count against the earnings threshold? A)Dividends from stocks. B)Rental income. C)Pensions and insurance annuities. D)Self-employment income.

D)Self-employment income. Rationale Earnings that count against the earnings threshold include W-2 wages and net self-employment income. All the others are not earned income for Social Security purposes.

All of the following statements are correct regarding a personal liability umbrella policy except: A)The PLUP provides protection above and beyond the liability limits of your homeowners and automobile insurance policies. B)The PLUP requires the insured to carry certain underlying minimum amounts of liability for homeowners and PAP. C)The PLUP insurer provides legal defense to the insured. D)The PLUP is only appropriate for individuals with a high net worth.

D)The PLUP is only appropriate for individuals with a high net worth. RationaleFuture earnings can be garnished. A PLUP may protect against garnished wages.

Under the HO-3 policy, all open-perils are covered, with some exceptions. All of the following are perils that are excluded from a HO-3 policy, except: A)Termite damage. B)Flood. C)Earthquake. D)Tornado.

D)Tornado. Rationale Termite damage is slow, so it's excluded. Flood and earthquake are definitely excluded. Wind, as in tornado, is covered.

Which of the following losses would be covered by a HO-3 policy? A)As a result of the overflow of a nearby lake, water enters the insured's basement and does $900 damage to floor coverings and the foundation. B)During a severe rainstorm, water enters through the wall of the basement and damages stored items worth $450. C)The insured, while having a party with many people present, discovers that excessive use of the sanitary facilities has resulted in the backup of water through the sewer, causing more than $2,250 in damage. D)While filling the bathtub, the insured's daughter answers the telephone, chats for some 20 minutes, and then discovers that the tub overflowed. The excess water seeps through the bathroom floor, and its weight causes the kitchen ceiling to collapse. Damages exceed $3,500.

D)While filling the bathtub, the insured's daughter answers the telephone, chats for some 20 minutes, and then discovers that the tub overflowed. The excess water seeps through the bathroom floor, and its weight causes the kitchen ceiling to collapse. Damages exceed $3,500. Rationale The water damage was accidental and was not among the excluded causes of water damage. Option a is flood damage, which is not covered. Option b is excluded because the water seeped through the basement wall. Option c is not covered because the backup of water through the sewer is not covered.

A worker's AIME: A)Must be determined by converting actual earnings into current dollars through an indexing factor. B)Is determined from wage information over prior years' work. C)Uses the highest 35 years of indexed earnings (for workers that worked at least that long). D)Yields an average amount of monthly earnings for all indexed years. E)All of the above.

E)All of the above.


संबंधित स्टडी सेट्स

eoc 2013 and 2014 released combined

View Set

Revised Chapter 1 Personal Finance Review

View Set

Ch. 10 Learnsmart Chromosome Organization & Molecular Structure

View Set

Business Chapter 5: How to Form a Business

View Set

Modern Database Management - Self Check 11

View Set

PS 160 Unit 2: Sound and Hearing, Temperature and Heat, and Thermal Properties of Matter

View Set